Day 12 Flashcards

1
Q

A 3 year old girl presents to A&E after her mother saw her have a ‘fit’.

The child was running around and hit her head on their coffee table.

Her mother reports that after hitting her head, she fell to the ground, turned pale and her arms and legs shook violently.

This lasted for about 10 seconds before her daughter regained consciousness and turned pink again.

She has otherwise been well recently and has no significant past medical history.

On examination, the child appears well and is now contently playing with some blocks in the waiting room.

What is the most likely cause of this episode?

A

Vagal stimulation

This episode of a generalised tonic-clonic seizure after head trauma with rapid recovery and no post-ictal confusion is consistent with a reflex anoxic seizure.

A reflex anoxic seizure is caused by overactivity of the vagus nerve, causing vasodilation and a collapse from a temporary reduction in cerebral perfusion.

How well did you know this?
1
Not at all
2
3
4
5
Perfectly
2
Q

A 12 year old girl is seen for her annual review in renal clinic following a diagnosis of polycystic kidney disease when she was 2 years old. She has hypertension which is successfully managed using ramipril. Her routine bloods are taken in clinic and her estimated glomerular filtration rate comes back as 50ml/min.

What is the stage of her chronic kidney disease?

A

Stage 3

The estimated glomerular filtration rate range for stage 3 chronic kidney disease is 30-59ml.

How well did you know this?
1
Not at all
2
3
4
5
Perfectly
3
Q

A mother brings in her neonate to the GP for her 6-week check.

The mother has a past medical history of bipolar disorder, and gave birth via spontaneous vaginal delivery at 38 weeks with no complications.

The GP hears a pan-systolic murmur on auscultation and so is referred for an echocardiogram.

Cardiac echocardiogram shows;

  • an enlarged right atrium
  • a small right ventricle
  • tricuspid incompetence

What is the most likely diagnosis?

A

Ebstein’s anomaly

The mother suffers from bipolar disorder and so is most likely taking Lithium monotherapy which, in pregnancy, can cause a congenital heart defect known as Ebstein’s anomaly.

This is characterised by;

  • an enlarged right atrium
  • a small right ventricle
  • tricuspid incompetence
How well did you know this?
1
Not at all
2
3
4
5
Perfectly
4
Q

A neonate on the intensive care unit has dysmorphic features, difficulty feeding and is not gaining weight.

He desaturates when feeding and his breathing at rest in noisy.

He is being nursed prone and is fed via a nasogastric tube.

The consultant diagnoses the Pierre Robin sequence.

Which of the following best describes the Pierre Robin sequence?

A
  • Cleft palate
  • retracted tongue
  • small lower jaw

This makes feeding infants difficult.

Specialised feeding equipment is used before surgical repair.

How well did you know this?
1
Not at all
2
3
4
5
Perfectly
5
Q

A father presents to the GP surgery with his six year old son.

The boy is shy and embarrassed but tells you after careful prompting that he is wetting the bed nightly.

You discuss the situation, and after ruling out organic causes, explain that bedwetting is a common phenomenon.

You learn that the boy has been using a star chart for the last month or so.

What would be the appropriate next step in management?

A

Enuresis alarm

An alarm that wakes the child to alert them to bed wetting, allowing them to pass urine in the toilet, is a useful addition to a star chart. The family already treat the bedwetting in a matter of fact and non-judgemental way. The boy is old enough to assist his parents in cleaning up the mess.

How well did you know this?
1
Not at all
2
3
4
5
Perfectly
6
Q

A 14 year old girl as admitted following an overdose.

She loses consciousness during the clerking and has no pulse.

CPR is commenced. The ECG shows the rhythm below.

What does the ECG show?

What is the most appropriate course of action?

A

Defibrillation

The rhythm shown is ventricular fibrillation, which is a shockable rhythm.

Evidence indicates the two biggest factors enhancing the survival of patients who have arrested is good quality compressions and prompt defibrillation if they have a shockable rhythm.

How well did you know this?
1
Not at all
2
3
4
5
Perfectly
7
Q

A 13 year old boy presents to the GP because his mum is worried that he has recently been ‘going blue’.

Friends and family have commented that he looks a bit blue.

On questioning, he reports feeling a little short of breath when playing football, but otherwise reports feeling well.

His mother says he has no known conditions apart from a hole in the heart that was mentioned early on but was never followed up on.

On examination, he appears moderately cyanotic.

He has clubbing of the fingernails.

There is a harsh holosystolic murmur loudest at the lower left sternal edge. The lung fields are clear to auscultation.

His oxygen saturations are 93%, respiratory rate is 18, heart rate is 80, and temperature is 36.8.

What is the most likely underlying cause of this patient’s colour change?

What is the diagnosis?

A

Reversal of direction of cardiac shunting

Eisenmenger syndrome describes the reversal of a left-to-right shunt (patent ductus arteriosus, atrial septal defect, or ventricular septal defect) to a right-to-left shunt.

How well did you know this?
1
Not at all
2
3
4
5
Perfectly
8
Q

A newborn is noted to have an intermittent squint at the newborn baby check.

What is the most appropriate management?

A

Refer if still present at 8 weeks

This is the recommendation for a squint in a newborn.

Many children have a squint in the first few weeks of life as the muscles are still weak.

However, most babies grow out of it quickly.

A squint persisting beyond 8 weeks is suspicious and treatment and investigations need to start as soon as possible to ensure the child does not develop amblyopia.

How well did you know this?
1
Not at all
2
3
4
5
Perfectly
9
Q

A newborn baby on the neonatal ward has difficulty feeding.

You notice that the baby is constantly drooling saliva and coughs up breast milk after feeding.

The pregnancy was complicated by polyhydramnios. The mother is a 38 year old primip.

On examination of the baby, there is upslanting palpebral fissures and a sandal toe gap.

What is the most appropriate initial investigation to determine the acute diagnosis?

A

NG tube and a chest X-ray

This is the diagnostic investigation to confirm oesophageal atresia.

The NG tube is passed down until it cannot be advanced any further.

An X-ray is taken at this point.

The tip of the NG tube is radio-opaque so the x-ray will show the level of the atresia.

How well did you know this?
1
Not at all
2
3
4
5
Perfectly
10
Q

A previously well 8 year old girl is seen at the General Practitioner following five days of dysuria.

She is otherwise well, with normal observations and examination.

Urine dipstick results are as follows:

  • Leuks: ++
  • Nitrites: ++
  • Hb: -
  • Ket: Trace

She is commenced on antibiotics.

What additional investigation should this patient have?

A

Urine microbiology

All urinalyses indicative for a urinary tract infection (UTI) should be sent for microscopy, culture and sensitivities. This will help inform both current and potentially future treatments. It also is advised in the NICE guidelines for UTIs in children.

How well did you know this?
1
Not at all
2
3
4
5
Perfectly
11
Q

What is the purpose of a DMSA Scan?

(2)

A

This is a scan to check for renal scarring.

It is especially pertinent in very young children with developing kidneys or those with recurrent urinary tract infections (UTIs).

How well did you know this?
1
Not at all
2
3
4
5
Perfectly
12
Q

What is the purpose of a Micturating cystourethrogram (MCUG)?

(2)

A

A micturating cystourethrogram is a form of functional imaging that looks urine flow and bladder function.

It might be indicated in some children with recurrent urinary tract infections (UTIs) or suspicions of overactive bladders.

How well did you know this?
1
Not at all
2
3
4
5
Perfectly
13
Q

An 18 month old boy is brought to A&E after his father saw him ‘have a fit’.

He has had a fever up to 39.0 degrees for one day, along with a runny nose and cough.

His father says his son was crying and then lost consciousness and started rhythmically jerking his limbs for around two minutes.

The boy regained consciousness shortly after but has been miserable and lethargic since.

Nothing like this has ever happened before, and the boy is normally fit and well.

What is the chance of something like this happening again in future?

A

30-40%

This boy with a short history of an upper respiratory tract infection with a high fever has had a febrile convulsion.

Febrile convulsions (or febrile seizures) are relatively common, occurring in around 3% of children.

Once a child has had one febrile convulsion, although the risk of epilepsy is low, the risk of having a future febrile seizure is high, around 30-40%.

How well did you know this?
1
Not at all
2
3
4
5
Perfectly
14
Q

A 5 year old boy presents to A&E with vomiting and lethargy.

His mum says he has been feeling feverish since he woke up this morning. He complained of a headache all day at school, and she had to collect him at lunchtime as he vomited. Since then, he has rapidly got worse and complained that his neck felt stiff and that the light hurt his eyes. She brought him in when she found a purple rash on his leg that stayed the same when she put a glass against it.

In the department, the boy appears unwell and drowsy. His respiratory rate is 36 and saturations are 99% in air. His heart rate is 60, blood pressure is 170/90 and capillary refill time is 3 seconds. His temperature is 39.0 degrees. His pupils are dilated and poorly reactive. There is a purpuric rash on his left ankle. Flexion of the neck causes involuntary flexion of the knees.

After your ABCDE Assessment and contacting a senior, what is the most important next step in managing this child?

A

Start IV antibiotics

This child with a headache, fever, vomiting, purpuric rash and photophobia, along with a positive Brudzinski’s sign (neck flexion triggers involuntary knee/hip flexion), has bacterial meningitis. This child is extremely unwell and has signs of raised intracranial pressure. The most important next step for this child is IV antibiotics.

How well did you know this?
1
Not at all
2
3
4
5
Perfectly
15
Q

A 7-year-old boy is brought into A&E via ambulance after falling off his bike. His parents report that he hit his head on the pavement and was unconscious for around 1 minute. He has vomited three times in the ambulance and once in A&E, and has remained drowsy since his fall. On clinical examination, there is extensive bruising over the right mastoid process, but no other signs of trauma. There is no focal neurology and his GCS is calculated to be 13.

Which of the following is the next best step in this child’s management?

A

CT Head within 1 hour

This is the correct answer. This child has vomited more than 3 times, has a reduced GCS and demonstrates signs of a basal skull fracture (bruising over the mastoid process) following a head injury, which are all worrying features that warrant a CT scan within the next hour. CT heads are useful for assessing the impact of trauma and guiding management.

How well did you know this?
1
Not at all
2
3
4
5
Perfectly
16
Q

A 4 year old boy is found collapsed on the floor.

He is unresponsive and does not have a pulse.

Which rhythm is the resus team most likely to find on his ECG?

A

Asystole

Asystole is the most common finding at paediatric cardiac arrests. It is not entirely clear why.

However, there is speculation that this is the most common rhythm as a result of respiratory causes being the most common cause of paediatric arrest, with hypoxia causing profound bradycardia and asystole in severe cases.

How well did you know this?
1
Not at all
2
3
4
5
Perfectly
17
Q

A 27 day old neonate is brought in to Accident and Emergency by his mother as she noticed that he is not feeding well.

The baby is exclusively breast fed and has not had any problems with latching on before.

The baby was last weighed a few days ago, and has been putting on weight.

On examination, the baby looks jaundiced and is lethargic.

The stool is of a normal colour and consistency.

The mother says that there have been fewer wet nappies over the last 12 hours.

There is some loss of skin turgor.

There is no hepatomegaly on palpation of the abdomen.

What is your most appropriate management?

A

Perform a full septic screen

This is correct because in a jaundiced unwell baby, sepsis is the first thing which has to be investigated due to the devastating consequences (organ failure, kernicterus, death). A urinary tract infection (UTI) is a common serious cause of jaundice in the newborn, and can often present with vague symptoms of lethargy or difficulty feeding. UTI in a newborn is particularly serious as it can rapidly progress to sepsis.

Normal neonatal jaundice occurs between 2-14 days

How well did you know this?
1
Not at all
2
3
4
5
Perfectly
18
Q

A 2-year-old child presents to paediatric A&E with her mother.

She has had difficulty breathing for the past two nights and has a barking cough.

On examination, there is no evidence of stridor or respiratory distress.

All other observations are normal.

Which of the following is the next best step in the management of this patient?

A

Give oral Dexamethasone

This is the correct answer. This patient has mild croup (evidenced by the lack of stridor and respiratory distress, and normal vital signs), which is managed with a single dose of oral Dexamethasone 0.15mg/kg.

How well did you know this?
1
Not at all
2
3
4
5
Perfectly
19
Q

When can chickenpox patients go back to school?

A

When all lesions have crusted over.

This girl presents with features in keeping with chicken pox, caused by varicella zoster. The virus is contagious from 1-2 days before the rash appears, until all vesicular lesions have crusted over.

How well did you know this?
1
Not at all
2
3
4
5
Perfectly
20
Q

A 4 year old boy who was recently adopted from Peru is brought into the paediatric clinic following referral by his GP who is concerned about his delayed growth.

On examination the patient is noted to have muscle wasting in the gluteal region.

On questioning his mother reports that his stools are often very yellow and often take many attempts to flush.

He has also suffered from frequent chest infections for as long as she can recall. She has not noticed any other changes recently.

Given the most likely diagnosis what would be the most appropriate investigation to order next?

A

Sweat test

As ions cannot be reabsorbed from the sweat a high to very high concentration of chloride ions will be noted.

A positive sweat test is defined a sweat chloride >60 mmol/L [b %60 mEq/L]. A positive result should prompt immediate referral to a cystic fibrosis specialist.

How well did you know this?
1
Not at all
2
3
4
5
Perfectly
21
Q

A 4 year old boy is seen at the GP after developing a pruritic rash on his face which has persisted for 5 days.

It started near has nose and now has spread to both sides of his face.

The lesions are erythematous and slightly weepy.

Some have a golden crusting over them.

His observations are stable and he has a temperature of 37.8C

What is the appropriate course of treatment?

What is the diagnosis?

What is the causative agent?

A

Fusidic acid

This boy has impetigo,

which is most commonly caused by Staphlyococcus aureus.

An appropriate treatment would be topic fusidic acid.

Hydrogen peroxide 1% cream is currently considered first line but this is not an option here.

How well did you know this?
1
Not at all
2
3
4
5
Perfectly
22
Q

7 year old girl is brought into the GP practice by her parents who noticed that she had her first period last week.

Her mother reports that she started her own periods early at 10 years of age.

On examination the GP notices breast budding and sparse pigmented pubic hairs around the labia.

She is plotted in the 99th percentile for height on her growth chart.

Her parents confirm that she is otherwise doing well and has not had any problems in school.

What is the medical term for this phenomenon?

What would be the most appropriate next step for the GP to take at this stage?

A

Refer to paediatrics

This girl has clearly entered puberty at a very early age.

Whilst this is not unheard of and the most likely cause of precocious puberty is that she has simply entered normal puberty prematurely, such an early onset of puberty should prompt a referral to a paediatric endocrinologist in order to investigate a possible cause and initiate treatment.

How well did you know this?
1
Not at all
2
3
4
5
Perfectly
23
Q

A boy born at 31 weeks gestation is admitted to neonatal intensive care following a ventricular haemorrhage.

The boy is at risk of central apnoeas.

On the ward round, there is a pause in his breathing that is lasting 15 seconds.

What is the most appropriate course of action?

A

Physical stimulation

Physical stimulation is the first step that should be considered to see if this triggers the baby to start breathing again.

After that, one should move onto an airway manoeuvre and supplying artificial breaths on oxygen via a bag-valve-mask if in a hospital setting.

How well did you know this?
1
Not at all
2
3
4
5
Perfectly
24
Q

A 3 year old boy is brought to A&E by his mother who is very worried about him.

He has been lethargic for the last 24 hours, not wanting to eat or drink much and his mother says he looks pale compared to normal.

Prior to that he was well, with no cough, coryzal symptoms or sign of infection.

He has a history of type 1 diabetes mellitus, which is managed with a basal-bolus insulin regime, administered in three daily doses.

His blood sugars are well controlled.

On examination the boy is laid down on the bed, rousable to voice but closes his eyes again soon after.

There is no visible rash, heart sounds are normal and chest clear on auscultation.

He has some mild suprapubic tenderness but no guarding or rigidity of the abdomen. Bowel sounds are normal.

Vital observations: temperature 38.4 degrees Celsius, heart rate 120bpm, respiratory rate 24/min, oxygen saturation 98% on room air, blood pressure 100/70.

Which of the following options is the most appropriate initial management of his presentation?

What is the diagnosis?

A

Cephalexin

Cephalexin is a broad-spectrum antibiotic that can treat urinary tract infections.

Although this boy has a suspected urosepsis, it is most appropriate to administer a broad-spectrum antibiotic in the first instance, until identification of the source of sepsis can be confirmed.

How well did you know this?
1
Not at all
2
3
4
5
Perfectly
25
Q

A 32-year-old primigravida attends a routine GP appointment with her partner to discuss the risk of having a child with cystic fibrosis. Neither herself or her partner have been diagnosed with cystic fibrosis and they are yet to undergo any screening tests to identify whether they are carriers of the cystic fibrosis gene.

What is the risk of their first child being affected by cystic fibrosis?

A

1 in 2500

This is the correct answer. In Europe, each parent has a 1 in 25 chance of being a carrier for the cystic fibrosis gene. If both parents were to be carriers of the gene, then the risk that their child is affected by cystic fibrosis is 1 in 4, as it is an autosomal recessive condition.

1/25 x 1/25 x 1/4 = 1/2500

How well did you know this?
1
Not at all
2
3
4
5
Perfectly
26
Q

A 9 year old boy is brought to A&E for difficulty in breathing that started at a school football practice.

On arrival at the department, he is unable to complete sentences, has a respiratory rate of 44 and a diffuse wheeze is heard throughout the chest on auscultation.

He is treated with high flow oxygen, back-to-back salbutamol nebulisers, ipratropium bromide nebulisers, and IV magnesium sulphate.

After these treatments, his respiratory rate is 8 and his chest is quiet to auscultation. His blood gas shows a pH of 7.42, pCO2 of 4.9 and pO2 of 9.6.

What is the most appropriate next step in managing this patient?

A

Call anaesthetist immediately

This patient is having an asthma attack and is deteriorating quickly.

In this patient, a silent chest and a normalising pCO2 are signs of impending respiratory failure as the respiratory muscles are nearing exhaustion.

An anesthetist should be called immediately to help with this rapidly tiring patient, as they might require urgent intubation.

How well did you know this?
1
Not at all
2
3
4
5
Perfectly
27
Q

A mother is worried that her daughter’s development may be delayed. On assessment the doctor reports the following:

  • The child is able to follow commands and responds to conversation in 3 or sometimes 4 word sentences.
  • She responds when asked that her favourite colour is purple.
  • Her mother reports that she has a vocabulary of approximately 250 words.

If the child’s development were normal how old are they most likely to be?

A

3 years

A 3 year old would be expected to use 3 or 4 word sentences frequently and have a vocabulary of at least 200 words. They should also be able to name one colour and answer when asked their name, age and gender.

How well did you know this?
1
Not at all
2
3
4
5
Perfectly
28
Q

A 4-year-old girl is brought to paediatric A&E with sudden-onset difficulty breathing.

Her parents say she has not been herself recently and describe coryzal symptoms over the past 24 hours.

The family recently emigrated from Russia and the parents are unable to recall which vaccinations their daughter has had.

On examination, she is drooling and has a inspiratory stridor.

Her temperature is 40.0ºC, she has a heart rate of 80, a respiratory rate of 43, and her oxygen saturations are 89% on room air.

Give the likely diagnosis.

Give the treatment.

Name the causative agent.

A

Endotracheal intubation

This child has presented with the cardinal features (sudden onset inspiratory stridor, drooling, high fever) of acute epiglottitis, and her incomplete vaccination history increase the likelihood of this diagnosis, as this condition is quite rare in the UK because all children are vaccinated against Haemophilus influenzae, the most common cause of acute epiglottitis.

This girl is acutely unwell and requires a secure airway before they go into respiratory arrest, therefore this is the priority before anything else should be done.

Endotracheal intubation in this scenario is usually carried out by an anaesthetist.

How well did you know this?
1
Not at all
2
3
4
5
Perfectly
29
Q

A 38 year old primip with gestational diabetes mellitus (GDM) has gone into spontaneous labour at 29 weeks.

Following signs of foetal distress, she had an emergency caesarean section.

Two hours later on the neonatal ward, the baby has a respiratory rate of 70 breaths per minute.

On examination, the baby is having difficulty breathing with nasal flaring, grunting and intercostal recession.

The oxygen saturations are 93% on room air. The chest x-ray shows a diffuse ground glass pattern.

What is the most likely diagnosis?

A

Respiratory distress of the newborn

The stem of the question includes the risk factors for respiratory distress of the newborn:

  • prematurity
  • maternal GDM
  • Caesarean section.

This is a typical presentation with an onset within 4 hours of birth with tachypnoea and signs of respiratory distress (grunting, nasal flaring, intercostal recession).

How well did you know this?
1
Not at all
2
3
4
5
Perfectly
30
Q

A 6 week old baby boy is brought to A&E as a GP referral because he is still yellow.

He has been getting progressively more yellow for the last 3 weeks.

His mum says his poo has also changed from thick black when he was born to mustardy to chalky.

His wee has also turned a dark yellow.

Otherwise, he has been breastfeeding well and is growing along his centiles.

He was born at 39+6 by spontaneous vaginal delivery after a normal pregnancy.

Antenatal screening and scans showed no abnormalities.

On examination, the baby appears well.

His abdomen is soft and a liver edge is palpable 2cm below the costal margin.

Antenatal screening and scans were all normal.

Blood tests show a significantly raised ALT, ALP and bilirubin (predominantly conjugated).

What is the most likely underlying cause of this baby’s presentation?

A
How well did you know this?
1
Not at all
2
3
4
5
Perfectly
31
Q

In VCV what is the most appropriate response for the GP to give when advising about the incubation period concerning the son?

A

Up to 21 days

The rash in this case is chickenpox caused by the varicella zoster virus (VZV).

Patients are infectious up to 3 days before lesions appear and up until the last lesion dries.

The incubation period for chicken pox is up to 3 weeks.

How well did you know this?
1
Not at all
2
3
4
5
Perfectly
32
Q

A 4 year old boy is brought by his parents to the GP practice as they are concerned that there is discharge from his right ear.

For the last two days, he has been irritable and tugging on the outside of his ear. He has just recovered from a recent upper respiratory tract infection (URTI). He is afebrile and is alert and active.

On otoscopy, there is a perforated tympanic membrane on the right.

What is the most appropriate management?

A

Prescribe amoxicillin

Antibiotics are prescribed if there is a perforated eardrum in acute otitis media.

This is recommended as it suggests a more serious infection and antibiotics will reduce the number of days of illness.

How well did you know this?
1
Not at all
2
3
4
5
Perfectly
33
Q

A 5 year old girl is brought to A&E with a 6 day history of fever.

Her mother also notices the whites of her eyes look red and that her hands and feet are red and swollen with some skin peeling at her fingertips.

She has swollen neck glands and a red tongue.

What is the most appropriate management?

A

Aspirin and IV Ig

This child with >5 days of fever, conjunctivitis, erythema and edema of the hands, cervical lymphadenopathy and mucosal involvement (strawberry tongue) most likely has a diagnosis of Kawasaki disease. Kawasaki’s is a rare condition that is treated with aspirin and IV Ig.

How well did you know this?
1
Not at all
2
3
4
5
Perfectly
34
Q

An 11 year old with a 6 month history of a wheeze and nocturnal cough was confirmed to have a diagnosis of asthma following spirometry and BDR (bronchodilator reversibility) test, and commenced on salbutamol and an ICS.

Four months later she presents to the GP with poor asthma control.

She has been using her salbutamol at least 10 times a week and is regularly woken at night with coughing.

Which of the following would be the next most appropriate step in her management?

A

Check inhaler technique

Correct inhaler technique is very important in order to ensure adequate delivery of treatment, and should always be assessed prior to changing management to avoid unnecessary addition of treatments. Children should always have inhalers prescribed with a spacer device (and also face mask if <5 years) in order to improve delivery of medication.

How well did you know this?
1
Not at all
2
3
4
5
Perfectly
35
Q

A four year old girl has presented to the GP several times with sore throats and pyrexia in the last two months without a diagnosis. She now presents to the ED with a fever, generalised lymphadenopathy, mild hepatosplenomegaly and a new petechial rash. A full blood count shows anaemia, raised white cells and low platelets. Blood film shows blast cells. What is the most likely diagnosis?

A

ALL

This is the commonest leukaemia in children (80%). There is marrow infiltration, causing anaemia and thrombocytopenia. Leukaemic blast cells (abnormal lymphocytes) are seen on the blood film and bone marrow biopsy.

How well did you know this?
1
Not at all
2
3
4
5
Perfectly
36
Q

A 2 year old girl is brought in to A+E by her parents as they are concerned about her breathing.

She has a constant high-pitched sound on inspiration.

She looks toxic and is leaning forward.

She is drooling and has a temperature of 39.5 degrees Celsius.

She was born in South America and her family moved to the UK recently.

What is the most likely diagnosis?

A

Acute epiglottitis

Continuous stridor in a child who looks toxic and is drooling is indicative of acute epiglottitis.

Especially since there is a history of living abroad where she may have missed several important immunisations.

How well did you know this?
1
Not at all
2
3
4
5
Perfectly
37
Q

A 5 year old girl presents to the GP with a 2 day history of fever, runny nose and a rash.

The rash is blotchy and red, started on her face and since spread to her chest and abdomen only.

She has no cough or conjunctivitis.

She is otherwise healthy. She takes no medicines.

She has never received any vaccinations because her mother does not think vaccinations are safe.

On inspection, the child appears miserable but well. Her respiratory rate is 16, heart rate is 80 and capillary refill is less than 2 seconds.

Her temperature is 38.0.

She has a fine macular rash across her head and torso, sparing the limbs, and post-auricular lymphadenopathy.

If the mother was 10 weeks pregnant and caught her daughter’s infection, which of the following is the fetus at highest risk of developing?

A

Deafness

This unvaccinated child with a fever and rash should raise suspicion for a rare infection like measles or rubella.

Although many of the symptoms are nonspecific (fever, rash, coryza), she has some features (rash starting on face and spreading to trunk, post-auricular lymphadenopathy) which are more specific to rubella infection.

There is also a lack of the preceding cough and conjunctivitis that is characteristic of measles infection.

Although rubella infection is generally mild, infection in nonimmune pregnant women puts the foetus at risk for congenital rubella syndrome, which is characterised by cataracts, brain damage and deafness.

How well did you know this?
1
Not at all
2
3
4
5
Perfectly
38
Q

A 7 year old girl with a history of asthma is brought to the GP by her mother as she is unable to keep up with the sports activities in school.

She is also getting symptoms at night and this is affecting her ability to concentrate in school.

She is currently on a salbutamol inhaler when required and 50 micrograms beclomethasone inhaler twice a day.

What is the most appropriate next step in management?

A

Start Montelukast

According to BTS guidance, if asthma is uncontrolled in children over 5 on a paediatric very low dose of ICS (inhaled corticosteroid) as maintenance therapy, you should add a leukotriene receptor antagonist (LRTA) such as montelukast.

The other option would be a long-acting beta-agonist (such as Salmeterol)

How well did you know this?
1
Not at all
2
3
4
5
Perfectly
39
Q

A mother brings in her 9-year-old son to the GP saying he has been feeling unwell, with a fever for the past 24 hours.

She reports he had tummy pain all day yesterday and vomited last night, and refused to eat breakfast this morning.

He is drinking normally and passing urine, and had one episode of loose stools today, which did not contain blood.

On examination, the child is pale.

Abdominal examination reveals generalised tenderness with guarding and tenderness to percussion.

Genital examination is unremarkable. Urine dipstick is normal.

His observations are: heart rate 130, blood pressure 80/50, oxygen saturation 98% on air.

Which of the following is the most likely diagnosis?

A

Appendicitis

The patient likely has appendicitis, and his observations (tachycardia, hypotension) suggest his appendix may have ruptured.

Appendicitis in children often presents with central abdominal pain (which later radiates to the right iliac fossa), vomiting and there may be a low-grade pyrexia.

The child is septic and needs to be taken to A&E for immediate treatment (intravenous fluids and antibiotics initially) which will likely involve surgery.

How well did you know this?
1
Not at all
2
3
4
5
Perfectly
40
Q

On the post-natal ward, a two-hour old baby boy born by spontaneous vaginal delivery at 38 weeks chokes whilst feeding.

His mother was giving him a bottle for the first time, and after about 30 seconds the baby started coughing, choking and vomiting milky white fluid.

The baby turned blue during this episode.

Now, 30 minutes later, he is pink in air but tachypnoeic with saturations of 96% and tachycardic.

An NG tube is inserted and the position checked with a chest X-ray which shows the tube is unable to move beyond an upper oesophageal pouch.

Which of the following is the most likely underlying cause of this choking episode?

A

Blind-ending oesophagus

Oesophageal atresia is a congenital disorder where the oesophagus fails to develop properly.

How well did you know this?
1
Not at all
2
3
4
5
Perfectly
41
Q

A six-year old girl presents to ED with her mother, who is worried.

Over the last week and a half the child has had a fever with a headache.

In the last two days the child has developed diarrhoea and a red rash covering her cheeks that spares the nasolabial folds.

She is also complaining of pain in her knees.

The child has been otherwise well, and has completed her vaccines.

What is the most likely diagnosis?

A

Parvovirus B19 infection

This is a very typical presentation of parvovirus B19 infection.

It is a DNA virus spread predominantly through respiratory secretions.

In children it causes fifth disease.

The ‘slapped cheek’ rash is typical, and shows sparing periorally and on the forehead.

It causes an initial viral prodrome.

In patients with disorders of the red blood cells it can cause an aplastic crisis.

How well did you know this?
1
Not at all
2
3
4
5
Perfectly
42
Q

A 2 week old baby boy, born prematurely at 29 weeks gestation, is seen during ward round on the neonatal unit.

He was delivered by emergency C-section for reduced foetal movements. He was born in fair condition with APGAR scores of 4, 5 and 6 at 1, 5 and 10 minutes of life. The baby was transferred to the neonatal unit and required two days of intubation and two doses of intra-tracheal artificial surfactant. There were no concerns in the rest of the pregnancy, and all antenatal scans were normal. He is being fed with fortified maternal expressed breast milk via syringe, and has good urine output.

On inspection, the baby is pink in air and has vigorous spontaneous movements. His chest is clear and abdomen is soft and non-tender without organomegaly. He is afebrile and his observations are within normal ranges. His weight is between the 2nd and 9th centile, and his head circumference is on the 95th centile. His white cell count is 3.0 and CRP is <0.6.

What is the most suitable next investigation in this baby?

A

Cranial ultrasound

This premature baby with a huge disparity between centiles for head circumference (very large) and weight (very small) most likely has hydrocephalus (buildup of CSF). As the suture lines of the skull are not fused at this stage, the head circumference can rapidly increase as the skull bones move apart to accommodate the excess fluid. The best option to investigate hydrocephalus is with a transcranial ultrasound scan. This premature baby may have had an intraventricular haemorrhage, leading to hydrocephalus.

How well did you know this?
1
Not at all
2
3
4
5
Perfectly
43
Q

A junior doctor is called to review a neonate that was born earlier today at term by spontaneous vaginal delivery.

The mother is concerned about a swelling on the baby’s head.

On examination there is a 3cm x 4cm central swelling on top of the baby’s head, which crosses suture lines.

The swelling has a border and is a normal colour.

The neonate is well otherwise.

Which of the following is the most likely diagnosis?

A

Caput succedaneum

Caput succedaneum is a collection of fluid that collects due to pressure on the head during vaginal delivery. (C)aput (s)uccedaneum (c)rosses (s)uture lines. This will usually spontaneously resolve within days.

How well did you know this?
1
Not at all
2
3
4
5
Perfectly
44
Q

Characterisitics of Cephalohaemotoma

(2)

A

A cephalohaematoma is a haemorrhage which does not cross suture lines and is periosteal.

This is benign and self resolves within the first few months of life.

How well did you know this?
1
Not at all
2
3
4
5
Perfectly
45
Q

A 1-year-old child presents to the GP with her parents who are concerned because she has been unwell for the past 4 days.

They say for the first 3 days her temperature was 38.4C, and this has come down since then and she is now afebrile.

They are now worried because since yesterday they have noticed a rash on her chest, with minimal spread.

On examination there is cervical lymphadenopathy and a maculopapular rash on the child’s chest.

What is the diagnosis from the picture?

Which of the following is the most likely causative organism of this child’s condition?

A
  • roseola infantum
    • Human herpes virus 6

A 3-5 day fever followed by a maculopapular rash is characteristic of roseola infantum, a condition caused by human herpes virus 6.

How well did you know this?
1
Not at all
2
3
4
5
Perfectly
46
Q

A 4-week-old baby girl is brought into A&E by her parents as they are concerned that her skin is yellowing.

On further questioning, the parents report that she has been feeding well and growing along her centiles, but her stools have recently become very pale.

She was born at term by spontaneous vaginal delivery and all antenatal screening tests came back as normal.

Blood tests reveal: ALT 50 U/L, ALP 873 U/L, total bilirubin 135 mmol/L, conjugated bilirubin 110mmol/L. On abdominal examination, masses can be palpated in both the left and right upper quadrant.

What is the most likely diagnosis?

A

Biliary Atresia

Biliary atresia is a paediatric condition where the bile ducts become progressively fibrosed and obliterated, which obstructs the flow of bile and presents as cholestasis in the first few weeks of life.

Abnormally high levels of conjugated bilirubin is a characteristic finding in biliary atresia.

The spleen becomes palpable in the 3rd-4th week of life (LUQ mass), with the liver also becoming enlarged (RUQ mass).

Prolonged neonatal jaundice with a raised conjugated bilirubin fraction, is biliary atresia until proven otherwise.

How well did you know this?
1
Not at all
2
3
4
5
Perfectly
47
Q

A six year old girl presents to the GP with a rash. The rash has been present for around one month, and comes and goes.

It is itchy and erythematous, and affects the flexures.

She has had no febrile illnesses recently and developed normally.

The child is otherwise well, and only uses a preventer inhaler and antihistamines for hayfever.

What is the most likely diagnosis?

A

Atopic dermatitis

The history and examination findings suggest atopic dermatitis (eczema). In children it typically affects the flexures (antecubital and popliteal fossae, and neck). It is associated with hayfever, other allergies and asthma. It may resolve during puberty, a minority represent in early adulthood.

How well did you know this?
1
Not at all
2
3
4
5
Perfectly
48
Q

A young woman attends a GP surgery with her newborn son for a routine check-up.

The boy was hypotonic at birth, has Brushfield spots in the iris, and is known to have a congenital heart defect.

What is the most likely diagnosis?

A

Trisomy 21

Hypotonia at birth, congenital heart defects (occurring in 40%) and Brushfield spots are features of trisomy 21 (Downs). Brushfield spots are small, white-grey aggregations of connective tissue at the peripheral iris.

How well did you know this?
1
Not at all
2
3
4
5
Perfectly
49
Q

A 6 year old boy presents to the GP with his mother.

Over the last 48 hours he has had a sore throat, headache and fever.

In the last 24 hours he has developed an erythematous rash over his face and torso.

He was previously well and is up to date with all scheduled immunisations.

On examination the rash is coarse to touch and blanches under pressure.

His tongue appears bright red but there are no ulcerations.

There are no lesions inside his mouth or elsewhere on his body.

What is the most likely cause of his symptoms?

A

Scarlet fever.

This boy presents with features in keeping with scarlet fever, which initially presents with non-specific features such as headache, fever, sore throat.

The characteristic rash is described as ‘sandpaper’ texture and children can develop ‘strawberry tongue’, named as the tongue appears bright red.

50
Q

A 7 year old boy is brought to paediatrics clinic for a review.

He has known chronic problems with cough, wheeze and recurrent respiratory infections.

He does active breathing and other chest physiotherapy techniques with his mum every day, and coughs up large volumes of sputum.

He was born at term by spontaneous delivery, but stayed in hospital for 5 days as he was late to pass meconium.

As a toddler, he had poor weight gain and loose stools.

He takes dornase alfa and creon.

He has no allergies.

Everyone else in the family is well.

On inspection, the boy is short and skinny for his age.

Which of the following blood tests would be raised at birth in this child?

A

Immunoreactive trypsinogen

This child, with chronic respiratory disease including bronchiectasis (coughing up large volumes of sputum) most likely has cystic fibrosis.

This case highlights the different ways cystic fibrosis can present in childhood:

  • meconium ileus as a neonate,
  • failure to thrive
  • loose stool as a toddler
  • short stature
  • and chronic respiratory disease in older childhood.

At birth, the heel prick test screening for cystic fibrosis shows a high immunoreactive trypsinogen level.

51
Q

A newborn, born through breech, has a positive Barlow and Ortolani test on screening. What is the most useful next imaging modality given the likely diagnosis?

A

Ultrasound scan

This is a un-invasive technique with no radiation associated. The lack of ossification at this age makes ultrasound the most useful imaging to assess hip joint integrity.

52
Q

You are an FY2 working on paediatrics and are called to assess a 2-hour old baby boy who vomited green fluid immediately after his first feed.

The baby was born by a spontaneous vaginal delivery at 37 weeks after a pregnancy complicated by polyhydramnios.

The baby’s apgar scores at 1, 5 and 10 minutes were 8, 9 and 10 respectively.

On inspection, the neonate has a single palmar crease and a wide gap between the big toe and second toes bilaterally.

An abdominal x-ray is performed which shows a distended stomach and distended duodenum, which are separated by the pyloric valve.

Which of the following genetic changes is most likely present in this baby?

A

Trisomy 21

This baby with bilious green vomiting immediately after the first feed, a ‘double bubble’ on abdominal X-ray, a sandal gap in the feet and a single palmar crease most likely has duodenal atresia associated with Down’s syndrome.

53
Q

A two-month-old is seen in the paediatric emergency department after having two temperatures above 39.0C.

He is lethargic and off his feeds. As part of a septic screen, urinalysis and cultures have been requested.

What is the most appropriate method for obtaining a urine specimen for these investigations?

A

Urine collection pad

Collection pads provide a practical, reliable and non-invasive method of collecting urine for children who cannot voluntarily provide a sample or aren’t potty-trained. Urine can then be quantified and also separated for analyses.

54
Q

A 10-year-old boy is brought into A&E after his parents witnessed him having a ‘seizure’.

His parents report that he had been complaining of feeling light headed and sweaty before he collapsed to the floor and had a brief episode of jerking movements affecting all four of his limbs.

The episode lasted for around 30 seconds, after which he felt weak for a further 15 minutes.

He did not bite his tongue or become incontinent of urine.

There is no family history of epilepsy; however, his notes show that he had a febrile convulsion at 6 months of age and has fainted on multiple occasions.

He has no history of recent illness or fever and his latest ECG showed no abnormality.

Which of the following is the most likely cause of his collapse?

A

Vasovagal Syncope

The preceding history of feeling light headed and sweaty is classically seen during vasovagal episodes.

The most likely reason for his brief convulsion is hypoperfusion to the brain due to a sudden drop in blood pressure.

Cardiac arrhythmias can present in a similar manner, but the fact that his ECG is normal is reassuring.

55
Q

What is a Reflex Anoxic Seizure?

(2)

A

Reflex anoxic seizures are paroxysmal self limited periods of asystole that are triggered by pain, fear or anxiety.

During these episodes the child becomes extremely pale, there may also be clonic movements, upward eye deviation, urinary incontinence and rigidity.

56
Q

A neonate is born at 36 weeks by elective caesarean section.

Shortly after delivery, she develops tachypnoea and looks in distress.

There was no meconium staining of the liquor.

Chest x-ray shows hyperinflation of the lungs and fluid in the horizontal fissure.

What is the most likely diagnosis?

A

Transient tachypnoea of the newborn (TTN)

TTN describes tachypnoea shortly after birth, and often resolves within the first day of life with supportive therapy with Oxygen.

It is caused by delayed resorption of fluid in the lungs and is strongly associated with caesarean section and prematurity.

X-ray signs include hyperinflation, and fluid in the horizontal fissure.

57
Q

A 14 month old baby girl is brought to A&E because she has had diarrhoea for 3 days and vomiting for 1 day.

The diarrhoea has been 4 times a day and watery with no blood.

She has vomited yellow fluid 2 times today.

The parents are worried because she has been refusing all food and not drinking at all for the last 24 hours.

She has had 5 wet nappies today, which is 2 fewer than normal.

She is otherwise healthy and takes no regular medicines.

On examination, the child appears well and is playing.

Her skin is pink and warm with no mottling.

Her mucous membranes appear dry and her eyes do not appear sunken.

Her respiratory rate is 30 and saturations 99%.

Her heart rate is 110, blood pressure is 118/78 and capillary refill is 2 seconds centrally. Her temperature is 37.1 degrees.

Her abdomen is soft and non-tender.

A urine dip shows no abnormalities.

What is the most appropriate management for this child?

A

Dioralyte oral rehydration

This child is moderately dehydrated from a viral gastroenteritis.

Signs of moderate dehydration in this child include reduced urine output and dry mucous membranes.

Moderate dehydration without signs of shock can be initially managed with oral rehydration fluid like dioralyte, for example 50ml/kg to replace the deficit.

Reassuring signs in this child include pink colour, lack of mottling, lack of sunken eyes, and no tachycardia, tachypnoea or hypotension; if any of these signs were present, this would be a red flag for severe dehydration that would require more aggressive management.

58
Q

A 16 year old boy presents to A&E after he had a fit at school.

He felt fine in the morning when having breakfast. He suddenly collapsed in a lesson, went stiff and then his limbs jerked for about one minute.

He was confused afterwards.

He is normally well, takes no medicines and has no allergies.

No one in the family has epilepsy.

On arrival in A&E, he tolerates an oropharyngeal airway, his respiratory rate is 18 and saturations are 99% in air.

His capillary refill is less than 2 seconds, heart rate is 74 and blood pressure is 118/78.

His GCS is 8, pupils are equal and reactive and glucose is 5.8. He is afebrile and does not have a rash.

Visual field testing is normal. There are no focal signs on neurological testing.

A CT head shows a heterogeneous single mass in both the right and left frontal lobes.

Which of the following is the most likely diagnosis?

A

Glioblastoma multiforme

This teenager, with a first presentation of seizure with no personal or family history of seizures, is very concerning for a space-occupying lesion (SOL).

The description of a frontal mass on CT that crosses the midline (connecting the right and left frontal lobes via the corpus callosum) is most likely to be a grade 4 astrocytoma, known as glioblastoma multiforme.

59
Q

A 9 year old boy is brought to see his GP as his mother is worried about a rash she has noticed developing over the past 3 days.

She reports that he has been unwell with a chesty cough and fever for the past week and has been absent from school as a result.

On questioning the boy admits he has had some tummy pains and has experienced increasing difficulty getting up and down the stairs due to severe pain in his knees and ankles.

On examination his observations are normal and a non-blanching palpable purpuric rash is noted on the buttocks and shins.

What is the most likely underlying cause of this patient’s presentation?

A

Henoch-Schonlein purpura

Henoch-Schonlein purpura is the most common vasculitis of childhood and affects the small vessels.

The condition presents with a tetrad of:

  • rash
  • abdominal pain
  • arthralgia
  • glomerulonephritis
60
Q

Henoch-Schonlein purpura tetrad (4)

A

Henoch-Schonlein purpura is the most common vasculitis of childhood and affects the small vessels.

The condition presents with a tetrad of:

  • rash
  • abdominal pain
  • arthralgia
  • glomerulonephritis
61
Q

You are an FY2 doctor in paediatrics called to assess a 2-hour old baby girl who is turning blue.

She was born by elective Caesarean section at 39 weeks’ gestation.

The baby’s mother has type 2 diabetes managed with injectable insulins.

On inspection, the baby appears cyanotic with some respiratory distress despite supplemental oxygen via an oxygen hood.

On auscultation of the chest, the lung fields are clear.

Heart sounds S1 and a loud single S2 are heard, with no other sounds or murmurs.

You note from the observation chart that the baby is tachypnoeic, tachycardic and afebrile.

What is the most likely underlying cardiac defect in this patient?

A

Transposition of the great vessels

The most common form of congenital heart defect in infants of diabetic mothers is transposition of the great vessels.

62
Q

A 5-year-old boy is seen in the General Paediatric clinic with his parents after being referred by his GP for a 2 year history of constipation unresponsive to laxatives.

He passes stool once every 4-5 days with difficulty.

Despite struggling to pass stool he soils his underwear daily.

His mother states that he did not pass meconium until three days after birth.

On abdominal examination, the abdomen is distended and a faecal mass is palpable in the left iliac fossa. Digital examination reveals an empty rectum and tight anal sphincter.

The doctor suspects a diagnosis of Hirschsprung’s disease.

Which of the following investigations is most likely to confirm the diagnosis?

A

Rectal Suction Biopsy

Definitive diagnosis of Hirschsprung’s disease is established histologically by taking a biopsy at least 1.5 cm above the pectinate line, to demonstrate acetylcholinesterase positive nerve excess and the absence of ganglion cells.

63
Q

A 13 month old boy responds when he is called.

He waves goodbye and demonstrates stranger anxiety when first meeting the doctor.

He prefers to pick up toys with his right hand.

He is able to reach for objects and is able to say a few words.

His mother says that he is generally a happy child and is usually smiling and laughing.

He points with his finger at the pictures in the book.

He is unable to scribble or to draw a vertical line.

What is the most worrying observation?

A

Hand preference

Hand dominance normally develops by 2 years and children <18 months old should not have a hand preference.

This may indicate spastic hemiplegia or cerebral palsy.

64
Q

The parents of several young children with congenital heart defects are being taught paediatric basic life support.

To what depth should the parents be advised to aim for when performing chest compressions?

A

One-third of the chest depth

As babies and children are growing, relative lengths are used to guide compressions. The advised depth to aim for when performing chest compressions in children is roughly one-third of the chest’s depth.

65
Q

A 2 month old baby boy with DiGeorge syndrome is brought in to PAU by his worried mother due to his recent episodes of non-bloody and non-bilious vomiting which have been occurring regularly after most of his feeds.

He is predominantly formula fed having been weaned at 6 weeks.

The pregnancy and delivery were both uncomplicated and he was born at term.

He has continued to feed as normal and no change has been noted in his bowel movements.

He was born weighing 3.5 kg and now weighs 4.8 kg.

Given the presenting history what is the most likely diagnosis?

(2)

A

Subacute sclerosing panencephalitis.

  • This boy presents with features in keeping with measles infection.
  • Subacute sclerosing panencephalitis is a possible long-term complication, which can occur up to 10 years after initial measles infection.
66
Q

A 7 year old girl, who has been doing very well at school and had been physically well, presents to A&E with new onset seizures and her family have noticed that she does not seem to know where she is or what she is doing for some of the days of the last week and she cannot remember simple things.

She can switch from crying to laughing within the space of minutes for no apparent reason.

She has never had mental health problems and neither has anyone in her family.

She has no history of epilepsy or febrile seizures and regains full consciousness between seizures.

Physical examination shows some cuts to the right arm, which the child cannot explain, and fundoscopy reveals retinal haemorrhage.

What is the next most appropriate step?

A

Referral to children’s social care

There is a long list of differential diagnoses that may be causing this girl’s symptoms, including trauma, autoimmune encephalitis, space occupying lesion or non-convulsive status epilepticus.

This girl presents with new onset seizures, emotional lability and cognitive decline.

In the context of symptoms such as these, which may represent brain injury, retinal haemorrhage can be suggestive of non-accidental injury (commonly known as ‘shaken baby syndrome’, but seen in older children as well who may have sustained traumatic brain injuries as a result of physical abuse).

The unexplained cuts to the child’s arm add weight to the fact that this scenario should be escalated to children’s social care immediately.

67
Q

A sixteen-year old boy presents to ED with a five day history of fever, sore throat, swelling in the neck and pain in the abdomen.

He has recently started at sixth form college and had been otherwise well.

Two days previously he had visited his GP, who had diagnosed him with tonsillitis and started a course of oral amoxicillin.

He has since developed a widespread rash and is very worried.

What is the most likely diagnosis?

Why did he develop the rash?

A

Epstein-Barr virus infection

Epstein-Barr virus causes a mild illness in the immunocompetent. Features include lymphoedema, pharyngitis and hepato- or splenomegaly.

It is associated with certain lymphoproliferative disorders, such as Burkitt’s lymphoma.

When incorrectly treated with antibiotics, and most commonly amoxicillin and ampicillin, patients with EBV can develop a scarring rash.

68
Q

A 5 year old boy presents to the GP with his father. He has had a fever, runny nose and cough over the last few days and has now developed a red rash all over his body. His father is uncertain about which immunisations the boy has received. On examination the boy has an erythematous, blanching maculopapular rash all over, with grey spots on his buccal mucosa.

Which of the following options is the most likely pathogen responsible for his symptoms?

A

Measles virus.

This boy presents with features in keeping with measles infection.

69
Q

A 5 day old male baby was commenced on phototherapy treatment for hyperbilirubinaemia.

After 14 hours of treatment, serum bilirubin levels fall to 285 micromoles/L (treatment threshold for phototherapy in this group is 350 micromoles/L).

Which of the following is the most appropriate course of action regarding management of this baby’s jaundice?

A

Stop phototherapy and recheck bilirubin levels in 12-18 hours time. If bilirubin remains >50 micromoles/L below treatment threshold at this time, no further monitoring is needed.

After commencing phototherapy, bilirubin levels should be monitored regularly (4-6 hours after first starting and then 6-12 hours when levels are stable or falling).

Once bilirubin levels have fallen over 50 micromoles/L below treatment threshold, phototherapy can be stopped. Bilirubin should then be rechecked in 12- 18 hours to assess for rebound hyperbilirubinaemia.

If at this time levels remain >50 micromoles/L, no further monitoring is needed. However, if bilirubin has risen to within 50 micromoles/L below treatment, levels should be rechecked again in 12 hours (and consider restarting phototherapy).

70
Q

A 5 year old girl with Trisomy 21 is seen in the paediatric cardiology clinic.

She has no cardiac symptoms, and on auscultation, a systolic murmur is heard at the upper left sternal edge which radiates to the back.

There is fixed splitting of the second heart sound.

What is the most likely cause of her examination findings?

A

Atrial septal defect

ASDs and atrioventricular septal defects (AVSD) are a common type of congenital heart disease seen in patients with Down’s syndrome.

It causes an ejection systolic murmur at the upper left sternal edge.

The first heart sound is normal but there is wide fixed splitting of the second heart sound.

71
Q

A 15 year old girl attends her local GP with a one week history of fever, sore throat, myalgia and fatigue.

She has her GCSEs coming up and is worried about being too exhausted to sit the exams.

She reports that swallowing is really painful to the point where she hasn’t had much to eat or drink recently.

On examination, she has significant cervical lymphadenopathy.

A Paul-Bunnell (for mono) test is positive.

Which of the following is a potential complication of this condition?

A

Splenic rupture

This teenager with sore throat, flu-like symptoms, fatigue and cervical lymphadenopathy has glandular fever.

A rare but important complication of glandular fever is splenomegaly from white blood cell sequestration.

The enlarged spleen is at risk of rupture in severe cases.

72
Q

A 7 year old boy has been admitted to the PAU after 1 day of vomiting and diarrhoea.

His parents are worried that he may have been suffering from food poisoning.

He was admitted by the paediatric registrar as he wasn’t able to keep down sips of water.

The registrar has started him on suitable antiemetics and has organised a series of investigations in order to establish a possible cause.

You are asked to write up suitable full maintenance fluids for the next 24 hours.

On reviewing the notes you see that the boy weighs 27kg and there are no concerns about heart failure, DKA or recent trauma.

On examination his mucous membranes appear moist and his bloods do not show any electrolyte abnormalities.

What would be the most suitable fluids regimen to start him on for 24 hours full fluids maintenance?

A

1640ml of 0.9% NaCl + 5% dextrose given in bags of 500ml and made up with 10mmol KCl per 500ml bag

This is the correct choice of fluids routinely used in the paediatric setting. This option provides sufficient electrolytes and the volume has been calculated correctly.

So for a 27kg patient the total fluid volume would be 1000ml + 500ml + 140ml = 1640ml total.

73
Q

A 4-year-old boy presents to A&E with his parents as they are concerned that his face and eyelids have become swollen.

On further questioning, his parents report that over the past 3 days his urine has been frothy. He has no dysuria, frequency, or abdominal pain.

On inspection, there is significant periorbital and facial oedema with milder pitting oedema up to his shins bilaterally.

Abdominal examination is unremarkable.

Urine dipstick shows

  • protein +++, blood nil
  • leucocytes nil
  • nitrites nil.

Blood tests reveal: haemoglobin 13.4 g/dL, white cell count 10.9 x 10^9/L, albumin 20g/L, creatinine 103 μmol/L, urea 12 mmol/L.

Which of the following is the most likely diagnosis?

A

Minimal Change Disease

This child has presented with the classical triad of oedema (usually periorbital), proteinuria, and hypoalbuminaemia seen in nephrotic syndrome.

In children, the most common cause of nephrotic syndrome is minimal change disease (accounts for around 70% of cases).

74
Q

Minimal Change Disease triad

A

This child has presented with the classical triad of

  • oedema (usually periorbital),
  • proteinuria,
  • hypoalbuminaemia

In children, the most common cause of nephrotic syndrome is minimal change disease (accounts for around 70% of cases).

75
Q

Features of Alport syndrome

(3)

A

Alport syndrome is a rare genetic condition characterised by;

  • progressive kidney failure
  • abnormalities of the inner ear (sensorineural hearing loss)
  • eye (lens dislocation)

It typically causes a haematuric nephropathy

76
Q

An 8 year old boy presents to the GP with a runny nose, sore throat and cough that he has had for the past 2 days.

He reports feeling OK and asks if he can play football this afternoon, but his mother is worried. He is normally fit and well.

He appears well with a respiratory rate of 20 and saturations of 100%.

On ENT examination, his throat is pink with no swelling or exudate of the tonsils, and his tympanic membranes are visible with no bulging or effusion.

On auscultation of the chest, you hear a soft systolic murmur at the left sternal edge.

The murmur does not radiate and there is no thrill on palpation.

He has never been noted to have a murmur before.

What is the most appropriate management for this murmur?

A

Review again in 2 weeks

This patient is presenting to the GP with a common cold and a soft systolic murmur at the left sternal edge.

This is most likely to be an innocent murmur.

Innocent murmurs are very common in children (up to 30% of children at some point), and can be precipitated by febrile illness.

As this child currently has a mild viral infection, the best option is to review the murmur in a few weeks once the infection has resolved, to see if the murmur has disappeared with the infection.

The infection is likely to be a mild viral upper respiratory tract infection, as evidenced by a lack of tonsillar exudate, angry red throat and no sign of ear infection (bulging tympanic membrane with effusion).

77
Q

A mother brings in her 3-year-old child to paediatric A&E.

She reports that her daughter has had a sore throat for the past 2 days, but over the past few hours has become feverish and started drooling from her mouth.

The mother does not speak English very well and is unable to give a full immunisation history.

The child is pyrexial (38.4 C) and has saliva drooling down her chin. On inspiration, there is a soft, high-pitched sound. Her doctor suspects acute epiglottitis.

Which of the following is the most likely causative organism of this child’s condition?

A

Haemophilus influenzae type B

This child is presenting with acute epiglottitis, suggested by the high fever, drooling and soft inspiratory stridor.

It is most commonly caused by Haemophilus influenzae type B, and should be considered in children from overseas who may not be up to date on their immunisations.

Since the introduction of its vaccine, it is rare in the UK.

78
Q

A 5 year old boy presents to the GP with his father.

He has had a fever, runny nose and cough over the last few days and has now developed a red rash all over his body.

His father is uncertain about which immunisations the boy has received.

On examination the boy has an erythematous, blanching maculopapular rash all over, with grey spots on his buccal mucosa.

Which of the following options is the most likely method of transmission of his infection?

A

Coughing and sneezing.

This boy presents with features in keeping with measles infection, spread by respiratory droplet transmission.

79
Q

A 15 year old girl is brought to A&E unconscious after a seizure.

Her friends said she had been acting strangely and said her stomach hurt that morning.

She was sweaty in class, and then fainted and started jerking for a few minutes.

The girl is known to have type 1 diabetes mellitus.

Her mother arrives and says she saw her daughter inject her insulin that morning, but that she didn’t see her have any breakfast.

On examination, the girl is not responsive to pain.

Her respiratory rate is 16, heart rate is 100 and blood pressure is 98/54.

Her pupils are 3 mm diameter, equal and reactive to light.

What is the most appropriate definitive management at this stage in the emergency department?

A

2ml/kg IV 10% dextrose

This teenager with known Type 1 diabetes who missed a meal and experienced abdominal pain, strange behaviour, sweating, loss of consciousness and seizure is presenting with acute hypoglycaemia. As she has arrived in A&E, the most appropriate definitive management is an intravenous glucose solution.

80
Q

A 5 year old boy is brought to the GP by his mother.

He has been feeling generally unwell over the last 2 days with lethargy, headache and fever.

He was previously well and is up to date with all scheduled immunisations.

He has developed a rash on his torso.

Which of the following is the most important feature of this rash to determine in the first instance?

A

Blanching or non-blanching.

Any child presenting with a rash must be considered as a possible case of meningitis or meningococcal septicaemia in the first instance.

This presents with a rash, which is non-blanching under pressure and can be fatal if not treated early.

If the rash is non-blanching, other differentials can then be explored.

81
Q

A young mother with her first born son presents to the paediatric emergency department.

The child is six weeks old, listless and appears dehydrated.

The mother describes progressive vomiting immediately after feeding. Initially low volume, it has increased to large volume and projectile.

On examination, the child has sunken eyes and fontanelle.

On palpation of the abdomen there is a mass in the epigastric region.

Given the most likely diagnosis, what is the most appropriate management plan?

A

Correct electrolyte abnormalities before surgery (pyloromyotomy)

Often infants will have low potassium and low sodium from vomiting and dehydration.

This must be corrected.

There may be hypochloraemic alkalosis.

Definitive treatment is surgical, with division of the hypertrophied pylorus.

82
Q

A two month old girl is being investigated for faltering growth.

She was born at term with an uncomplicated antenatal and postnatal course.

Which sign would most be most apparent on examination to indicate she has congestive cardiac failure?

A

Hepatomegaly - other signs are difficuklt to detect in children; oedema, clubbing, JVP

The liver can reliably be palpated in babies and young children given how small they are.

Hepatomegaly occurs as a result of back-pressure from the congestion.

The degree to which its palpable can be documented in terms of centimetres, or whether just the liver edge may be felt.

This should be done as part of a paediatric cardiac examination.

83
Q

A father presents to the GP surgery with his 8 year old daughter.

He describes strange occasional episodes where the child suddenly stops moving completely and stares ‘into space’.

On further questioning you find that these episodes last around 10 seconds. The child is otherwise well and has no medical history.

She is unaware of these episodes, but gets embarrassed when they happen at school.

What is the likely diagnosis?

A

Childhood absence epilepsy

This typically occurs in girls aged 4 - 12 years. It accounts for 2% of childhood epilepsy.

84
Q

A 16-year-old girl presents to the GP with low mood. She says that she feels like she does not have much energy, is waking up early in the morning and is unable to concentrate at school.

She said she used to be part of the netball team, but recently has not been turning up to practice.

She has felt this way for a month and she is worried because it is affecting her studies and she has exams coming up.

Given the likely diagnosis of moderate depression, which of the following medications may help?

A

Fluoxetine

Fluoxetine is a selective serotonin reuptake inhibitor (SSRI) and is the only drug of its class licensed for use in patients under 18 years of age.

85
Q

A 3 year old boy is brought to A&E by his mother who is very worried about him.

He has been lethargic for the last 24 hours, not wanting to eat or drink much and his mother says he looks pale compared to normal.

Prior to that he was well, with no cough, coryzal symptoms or sign of infection.

He has a history of type 1 diabetes mellitus, which is managed with a basal-bolus insulin regime, administered in three daily doses.

His blood sugars are well controlled.

On examination the boy is laid down on the bed, rousable to voice but closes his eyes again soon after.

There is no visible rash, heart sounds are normal and chest clear on auscultation.

He has some mild suprapubic tenderness but no guarding or rigidity of the abdomen.

Bowel sounds are normal.

Vital observations:

  • Temperature 38.4 degrees Celsius
  • Heart rate 120bpm
  • Respiratory rate 24/min
  • Oxygen saturation 98% on room air
  • blood pressure 100/70.

Which of the following blood tests is most specific to bacterial sepsis?

A

Procalcitonin

Procalcitonin is an emerging blood test that shows specificity for inflammatory markers and can help detect bacterial sepsis. This boy’s presentation is in keeping with sepsis secondary to a urinary tract infection, but the source is currently unproven.

86
Q

A 16-year-old girl presents to the GP worried that she has not started her periods.

She has a past medical history of hypothyroidism and recurrent ear infections.

On examination, it is noted she has a webbed neck, low set ears and widely spaced nipples.

On auscultation, there is a systolic murmur loudest over the aortic valve.

Which of the following is the most likely diagnosis?

A

Turner’s syndrome

This is correct. Turner’s syndrome is a condition caused by an XO karyotype, and can result in delayed puberty.

Common features include short stature, webbed neck, widely spaced nipples, low set ears, wide carrying angles, hypothyroidism and recurrent otitis media.

There is also an increased risk of congenital heart defects, the two main ones being coarctation of the aorta, and bicuspid aortic valve (as in this case).

87
Q

A 8 year old girl has recently been diagnosed with leukaemia and the oncologists recommend starting her on chemotherapy.

Her mother disagrees and wants to try alternative therapies first. Her father wants to start the chemotherapy as soon as possible.

The father was married to the mother after the child was born.

The child understands the information given and is able to retain and communicate her decision, but is not able to weigh up the information given.

What is the most appropriate way to lawfully provide treatment?

A

The father can provide consent for treatment to be lawfully provided despite the mother’s lack of consent.

The law only requires doctors to have consent from one person in order to lawfully provide treatment. According to the BMA:

A father acquires parental responsibility if he is married to the mother at the time of the child’s birth or subsequently.

Therefore, in this scenario the father has parental responsibility as he was married to the mother after the girl was born.

88
Q

A 15 year old boy initially presented to GP with pain in his shins which was attributed to normal growing pains.

Following a further consultation in the practice the pain has not resolved and he has developed intermittent fevers and night sweats.

He has also developed a large amount of swelling in his left shin.

An initial plain film radiograph showed evidence of a lamellated type periosteal reaction in proximal portion of the left tibia with reduced bone matrix.

He is now being seen in a paediatric orthopaedics clinic for the results of magnetic resonance imaging and a bone biopsy.

Magnetic resonance imaging:

A mass is noticeable in the proximal left tibia which is approximately 4 cm in diameter. A small area of necrosis is visible at the centre. There is minimal evidence of cortical damage in the proximal tibia. No other areas of tissue hyper density or abnormalities are noticeable in the body.

Bone biopsy:

Histology: Small blue round cells are present and have a clear cytoplasm on haematoxylin and eosin staining.

Given his presentation and investigation results what is the most likely diagnosis?

A

Ewing’s sarcoma

Correct. Ewing’s sarcoma typically presented with the lamellated (onion skinning) periosteal reaction which is visible on x-ray. MRI typically shows a large mass with evidence of necrosis and on histology small blue round cells are visible with clear cytoplasms on haematoxylin and eosin staining.

89
Q

A 6 day old girl is brought to hospital by ambulance after her parents found her face had turned blue.

Her birth history was unremarkable.

Despite being on high-flow oxygen during the ambulance ride, her saturations remain at 82%.

Her airway and chest are clear on examination.

What is the most likely diagnosis?

A

Transposition of the great arteries

This is a cyanotic defect where the aorta and pulmonary arteries have their insertion points to the heart swapped around.

This results in two circuits running in parallel with a deoxygenated systemic circuit and an oxygenated pulmonary circuit.

In its purest form, this is not compatible with life.

However, usually there are several shunts in place (any combination of atrial septal defect, ventricular septal defect and patent ductus arteriosus).

In most cases, TGA is detected antenatally, or when presenting as cyanotic upon birth.

However babies can also maintain normal saturations if there is a large VSD (and thus no murmur) and patent ductus arteriosus (whose murmurs aren’t always heard until pressures build up) to compensate.

Thus when the ductus does physiologically close, the saturations can fall due to reduced right-to-left shunting.

Giving oxygen will not make a noteworthy difference as the pulmonary circulation is already saturated well.

90
Q

A neonate is delivered by emergency caesarean section at 32 weeks gestation after preterm premature rupture of membranes.

At 1 minute after birth, the neonate appears blue at the fingers and around the mouth and has a poor respiratory effort.

The baby is resuscitated, intubated and stabilized before transfer to the neonatal intensive care.

At 48 hours after delivery, the baby appears moderately cyanotic and is grunting on inspiration.

The baby has a raised respiratory rate, nasal flaring and subcostal recessions.

A chest X-ray shows a ground glass appearance to the lung field bilaterally.

Which of the following medicines given to the mother prior to delivery could help reduce the severity of this neonatal condition?

A

Dexamethasone

This preterm baby with signs of respiratory distress including cyanosis, grunting, tachypnoea, nasal flaring and subcostal recessions with a ground glass appearance on chest x-ray has neonatal respiratory distress syndrome.

If preterm delivery is suspected, maternal intramuscular steroid injection, such as dexamethasone, can help to boost foetal surfactant production and thus reduce the likelihood of developing neonatal respiratory distress syndrome.

91
Q

A mother brings her 5-year-old child to the GP because she thinks he is not developing like the other children in his class.

The child is microcephalic, has short palpebral fissures and no philtrum.

His mother does not recall any issues during her pregnancy.

What is the most likely cause of the child’s symptoms?

A

Maternal alcohol use

This child presents with features of foetal alcohol syndrome:

  • microcephaly
  • short palpebral fissures
  • hypoplastic upper lip
  • absent philtrum
  • reduced IQ
  • cardiac abnormalities
92
Q

After noticing an abnormality of the aorta and ventricles on ultrasound scans, a boy is delivered electively at a specialist unit.

An echocardiogram confirms his aorta is interrupted with his descending aorta connecting to his pulmonary trunk via his ductus arteriosus.

A pan-systolic is heard on auscultation.

Additionally, he is noted to have a cleft palate.

What genetic/chromosomal abnormality is this patient most likely to have?

A

22q11.2 deletion

The phenotype described is that of DiGeorge syndrome.

The cardiac involvement in people with DiGeorge syndrome is an inturrupted aortic arch.

They usually have a ventricular septal defect too.

Their underlying genetic mutation is a deletion on the q arm of chromosome 22.

93
Q

A 4 year old girl who recently started school is brought to her GP with a 24 hour history of feeling generally unwell, tiredness and headache.

She has been eating and drinking but less than normal. H

er mother states that she is usually happy and healthy but seems to have got gradually worse today.

No one else in her class has been unwell.

She has no past medical history and does not take any regular medications.

On examination the child looks unwell and appears quite withdrawn.

Vital signs: pulse 100bpm, BP 110/70, respiratory rate 22/min, oxygen saturations 98%, temperature 38.5 degrees C.

Systems examination is unremarkable.

She has cold hands and feet and a non-blanching purpuric rash on her left upper thigh.

Her GP refers her to the nearest hospital.

Which of the following treatments should her immediate family members receive?

A

Ciprofloxacin

This girl is highly likely to have meningococcal infection.

All household or close contacts should receive ciprofloxacin as prevention of spread of the disease.

A suitable alternative would be rifampicin.

94
Q

A junior doctor is asked to review a neonate on the postnatal ward.

He was born 10 hours ago and has suddenly turned blue and developed an increased work of breathing.

Upon examination, he has central cyanosis, he is not tachypnoeic, there are no additional breath sounds, and on auscultation of the heart there are no murmurs.

A chest x-ray shows cardiomegaly.

He has been given Oxygen by the nurses but this has not improved the cyanosis.

Which of the following is the next best step in the management of this patient?

A

Alprostadil

this child most likely has transposition of the great arteries - which is not compatible with life, however, he is likely being kept alive by a patent ductus arteriosus.

Alprostadil prevents the close of this duct

This is a case of duct dependent congenital heart disease, which is managed with prostaglandins.

This is to promote patency of the ductus arteriosus, which begins to close shortly after birth and is the cause of cyanosis in this situation.

This will allow the infant to survive long enough to allow investigations to confirm the diagnosis, which is most likely transposition of the great arteries, and thus surgical correction.

95
Q

A term baby on the neonatal ward has been delivered by an uncomplicated elective Caesarean section 3 hours ago.

The mother received betamethasone until 39 weeks.

The baby now has inter-costal recession and is having difficulty with breast feeding. The baby is alert and is afebrile.

The CXR shows hyper-inflated lungs and a fluid level.

What is the most likely diagnosis?

(3)

A

Transient tachypnoea of the newborn

  • Hyper-inflated lungs
  • and a fluid level in a newborn

is the characteristic presentation of transient tachypnoea in the newborn,

  • especially in caesarean section deliveries.
96
Q

A 7-year-old boy presents to the GP with his mother because she is concerned that he has lost weight.

His appetite has not changed, although he has been drinking a lot more than usual over the past 3 weeks.

On further questioning, his mother also reports that after being dry at night for the past year, one month ago he started wetting the bed again.

Both his mother and grandmother suffer from Graves disease.

Which of the following is the most likely diagnosis?

A

Type 1 Diabetes Mellitus

This child has presented with weight loss, polydipsia and polyuria which is in keeping with a diagnosis of diabetes.

Type 1 diabetes mellitus is the most common form of diabetes in children, and, combined with the family history of autoimmune disease (Graves thyroiditis), it makes it the most likely diagnosis.

97
Q

A 3-year-old boy presents to the GP with his parents, as they are concerned that he is sleeping more than usual.

On further questioning, they report that over the last 2 weeks he has experienced recurrent nose bleeds and bruising on his arms and legs.

There is no history of recent infection and symptomatic screen is otherwise unremarkable.

He has a known diagnosis of Down syndrome.

On clinical examination, he is febrile with a temperature of 37.9ºC and multiple lymph nodes can be palpated along the cervical chain and in the groin, along with notable testicular swelling.

Which of the following is the most likely diagnosis?

A

Acute lymphocytic leukaemia (ALL) is the most common childhood malignancy and often presents with

  • anaemia (fatigue and pallor)
  • thrombocytopenia (bruising and epistaxis)
  • neutropenia (recurrent infections and fever)

The testicular swelling is due to infiltration.

Individuals with a diagnosis of Down syndrome have a four-fold increased risk of developing ALL.

98
Q

You are working with the paediatrics team who are called to an emergency C-section at 37 weeks’ gestationn.

Immediately after birth, the baby lets out a strong cry.

As you towel the baby dry, the arms and legs resist extension, and the baby cries with stimulation. The hands and feet look a little blue, but the face and trunk are pink. The baby’s heart rate is 140 beats per minute.

What is the APGAR score of the baby?

A

9

99
Q

A 6-year-old boy is brought to A&E with a 7-day history of fever.

On further questioning, his mother reports her son has been complaining of aches and pains in his ankles and wrists and 2 days ago she noticed that his hands had become swollen with skin peeling off his fingertips.

On clinical examination, there is a polymorphous rash across his trunk and multiple enlarged lymph nodes are palpable along the anterior cervical chain.

Bilateral conjunctival erythema is also noted.

Given the likely diagnosis which of the following is the most appropriate management?

A

Aspirin and intravenous Immunoglobulin

This is the correct answer. This child with > 5 days of fever, conjunctivitis, truncal rash, adenopathy, oedema and desquamation of the hands most likely has a diagnosis of Kawasaki disease. This is treated with high-dose Aspirin and intravenous Immunoglobulin within 10 days of symptom onset. It is important to start treatment promptly to optimise patient outcomes.

100
Q

A baby boy is seen at the GP for his 6 week check.

He was noted to have a systolic murmur on his newborn check and is awaiting an echocardiogram.

On examination, the murmur persists with a new finding of weakened femoral pulses.

The rest of his newborn check is normal.

He is tracking well on his growth charts and his mother reports no issues.

What is the diagnosis?

If this defect is left untreated, which of the following complications is most likely to occur?

A

Cerebral aneurysms

The most likely diagnosis is coarctation of the aorta.

This is a narrowing of the aorta, most often after the left subclavian artery’s branches off the aorta.

Reduced perfusion pressures distal to stenosis trigger the renin-angiotensin-aldosterone system with limited effect as this won’t overcome the stenosis.

It however will increase perfusion pressures before the stenosis, including the carotids, resulting in higher cerebral perfusion pressures which can cause aneurysms to form.

101
Q

A 6 month old boy is brought to the GP by his parents as they are concerned that he has an obvious left scrotal swelling.

The boy is otherwise well in himself and there are no concerns with his development or growth.

He does not seem to be in pain and the parents think there has been no history of trauma to the area.

There has been no change to the number of wet nappies.

On examination, you can get above the scrotal swelling but are unable to clearly differentiate between the swelling and testes.

The swelling is trans-illuminable.

What is the most appropriate management?

A

Reassure the parents and advise them to return if there are any further problems

A hydrocele typically resolves in the first year of life and generally treatment does not start until >12months or if there are complications.

102
Q

A 36 year old woman who is 20 weeks into her pregnancy is diagnosed with cervical insufficiency and told that she is at risk of going into preterm labour.

For this reason, she is advised that she will be offered preventative steroids at 24 weeks gestation.

Which of the following conditions is the administration of antenatal steroids to protect against?

A

Respiratory distress syndrome

This is caused by inadequate production of surfactant, a substance produced in the lungs by type two pneumocytes which facilitates full expansion of alveoli by reducing surface tension. It is common in births <32 weeks as most surfactant is produced around 30 weeks gestation. Therefore, women at risk of pre term labour between 24 and 36 weeks are offered antenatal steroids in order to stimulate surfactant production.

103
Q

A 6 year old boy is on the paediatric ward.

He has been spiking temperatures for 5 days since admission with an unclear source.

His blood and urine cultures are negative, as is his lumbar puncture.

He has been commenced on IV broad spectrum antibiotics but his CRP has not improved.

He has developed a widespread maculopapular rash across his body.

The soles of his feet are swollen and erythematous and both his eyes are red.

Which phenomena would be consistent with the most likely diagnosis?

A

Peeling skin

This boy has Kawasaki disease. Desquamation occurs in the later stages of Kawasaki. By this point a diagnosis should have been reached. It classically occurs on the extremities but can occur elsewhere too.

104
Q

A 3-year-old girl presents to the GP with her parents.

They are concerned because of the past 4 days she has been suffering from coryzal symptoms and a fever.

On cardiovascular examination, a soft systolic murmur is audible over the left sternal edge.

No heaves or thrills are palpable and the apex beat is felt in the 5th intercostal space in the mid-clavicular line.

The murmur does not radiate and there are no added sounds.

Which of the following is the most likely diagnosis?

A

Innocent Murmur

This is the most likely diagnosis. This child has presented with a new onset asymptomatic systolic murmur following a febrile illness, which makes the diagnosis of an innocent murmur most likely.

Features of innocent murmurs include:

  • Localised with no radiation
  • May vary with posture
  • Soft systolic murmur with no diastolic component
  • No heaves or thrills
  • Asymptomatic
  • No added sounds

Febrile illnesses can accentuate innocent murmurs, and once the infection has resolved it is important to review the child to see if the murmur persists.

105
Q

A woman presents at week 10 of pregnancy to her GP for a booking visit.

As has been told by a friend who has recently given birth that all newborn babies are offered certain screening tests such as the heel- prick test and an examination at birth and 6 weeks, and wants to know if there are any other routine tests offered to newborn babies.

Which of the following options are also offered routinely as part of newborn screening in the UK?

A

Audiology

This is offered at 4-5 weeks after birth to all newborn babies in order to test for congenital causes of hearing loss, as hearing is key to development.

106
Q

A 5 day old baby boy is brought to GP for a routine blood spot test.

His mother has been told that this test is to check for conditions such as cystic fibrosis, but wants to know what else it tests for.

Which of the following conditions is also tested for in the blood spot test?

A

Sickle cell disease

This occurs in roughly 1:2,000 births.

It is a disorder characterised by abnormal haemoglobin which has the propensity to alter the shape of red blood cells into a ‘sickle’ shape, which can become stuck in capillaries leading to ischaemia and painful sickle cell crises.

107
Q

A boy is born at term requiring ventilatory support shortly after delivery.

He has low set ears, downward slanting eyes and noticeably appears to have a ‘twisted’ posture and extremities.

An ultrasound reveals multicystic, dysplatic kidneys.

Which of the following findings would be expected in the antenatal notes?

A

Oligohydramnios

The phenotype is classical of Potter sequence.

It arises from a common sequence of events caused by oligohydramnios.

The lack of amniotic fluid means there is a lack of ‘cushioning’ of the foetus, resulting in it getting squashed by the surrounding maternal organs - resulting in their characteristic appearance.

108
Q

A 10 year old boy is brought to A&E by ambulance.

His mother called 999 when he became short of breath at home, shortly after smoke from a neighbour’s bonfire had been drifting into the house through open windows.

The boy has a history of asthma but rarely needs to use his salbutamol inhaler. When he used it today, it did not have any effect.

He says his chest feels tight and he cannot complete sentences without pausing for breath.

He does not have pain anywhere.

Paramedics have given him nebulised salbutamol in the ambulance, which has helped the boy’s peak expiratory flow reading increase from 40% to 50% of that expected for his age.

Vital observations: BP 105/70, pulse 140bpm, respiratory rate 40/min, O2 saturations 96% on nebulised salbutamol without oxygen, temperature 37 degrees C.

On physical examination there is widespread bilateral wheeze audible both at the bedside and on auscultation.

Which investigation is most likely to confirm the diagnosis?

A

Peak expiratory flow rate which improves with bronchodilator treatment.

This boy presents with features of acute severe asthma. The improvement of peak expiratory flow rate with bronchodilator treatment is highly suggestive of acute severe asthma. If symptoms do not respond it is worth considering other differential diagnoses.

109
Q

What is the ‘Double bubble’ sign on radiography suggestive of?

A

The ‘double bubble’ sign is a feature of duodenal atresia.

It is seen on plain radiographs and ultrasound.

In duodenal atresia it is caused by a distended and air filled stomach and small bowel

110
Q

A baby boy is born via caesarean section at term and he is noted to be particularly floppy with a number of obvious physical abnormalities.

On examination you notice the following: brush field spots in the iris of both eyes, a single palmar crease in both hands, a flat nasal bridge and a small mouth with protruding tongue.

What is most likely to be the correct diagnosis in this case?

A

Down syndrome (trisomy 21)

Down syndrome is also known as Trisomy 21 as it involves a complete or partial third copy of chromosome 21 being present in the child’s genome.

This leads to the classic physical features described in the question stem, a diagnosis of Down syndrome may be suspected clinically based on the newborn’s appearance alone.

Further assessment by a senior paediatrician will then take place to confirm a diagnosis.

111
Q

A 4 day old baby girl born at 39 weeks is seen at home by a midwife.

Parents are concerned regarding a yellowish colouring of her skin which developed that morning.

The baby has otherwise been completely well.

The midwife has a transcutaneous bilirubinometer with her and suspects the baby is jaundiced.

Which of the following would be the next most appropriate action for the midwife?

A

Measure transcutaneous bilirubin levels with a bilirubinometer at home, and check serum bilirubin if bilirubin level is >250 micromoles/L

Transcutaneous bilirubin can be used to assess bilirubin levels in jaundiced babies who are over 35 weeks gestation and over 24 hours of age.

Furthermore, as the baby is otherwise completely well, there is no indication that she may need treatment or assessment in GP or hospital.

If transcutaneous bilirubin is found to be over 250 micromoles/L it should be corroborated with serum bilirubin levels and may require treatment.

112
Q

A 3 year old boy is brought to A&E by his mother who is very worried about him.

He has been lethargic for the last 24 hours, not wanting to eat or drink much and his mother says he looks pale compared to normal.

Prior to that he was well, with no cough, coryzal symptoms or sign of infection.

He has a history of type 1 diabetes mellitus, which is managed with a basal-bolus insulin regime, administered in three daily doses.

His blood sugars are well controlled.

On examination the boy is laid down on the bed, rousable to voice but closes his eyes again soon after.

There is no visible rash, heart sounds are normal and chest clear on auscultation.

He has some mild suprapubic tenderness but no guarding or rigidity of the abdomen. Bowel sounds are normal.

Vital observations:

  • temperature 38.4 degrees Celsius
  • heart rate 120bpm
  • respiratory rate 24/min
  • oxygen saturation 98% on room air
  • blood pressure 100/70

What is the most likely differential diagnosis?

A

Sepsis

Sepsis is a clinical syndrome, presenting with complications of the body’s response to infection. It is a leading cause of paediatric mortality. Sepsis in children can present in a number of ways, many of which are non-specific, as a result of systemic inflammatory response to suspected or proven infection. This boy presents with features of fever, poor oral intake, lethargy, altered mental status, with symptoms suggestive of a urinary tract infection which has led to sepsis.

113
Q

A 15 year old boy presents to GP with a 4 month history of coughing at night which often wakes him up, as well as a wheeze.

After taking a full history and examination, asthma is suspected and the decision is made to perform an investigation to confirm the diagnosis.

Which of the following is the best first- line investigation? (2)

A

Spirometry and BDR (bronchodilator reversibility).

This is the first line investigation for suspected asthma, and a positive result will confirm the presence of an obstructive airway pattern which is reversible by bronchodilator.

Spirometry is performed first, and if an obstructive pattern is noted, BDR is then performed to confirm the diagnosis.

Spirometry is a form of pulmonary function test which measures the speed and volume of inspiration and expiration.

In the diagnosis of asthma, FVC (forced vital capacity- the volume of forces exhalation after maximal inhalation) and FEV1 (Forced expiratory volume- the volume of air after 1 second of forces exhalation) are the key parameters used.

An obstructive pattern would have an FEV1:FVC ratio of less than 70%. The BDR test involves comparing spirometry results before and 15 minutes after bronchodilator inhalation.

An improvement of FEV1 of >12% indicates reversibility.

114
Q

A 3 year old girl has been febrile for 3 days.

She has become increasingly unsettled, but remains relatively active during this course.

She has now developed a rash on her face which has spread to her torso and limbs.

The rash is rough and maculopapular throughout.

On her face, it predominantly covers her cheeks and is not near her mouth – where her lips are bright red.

She is noted to have palpable cervical lymph nodes.

She is not eating or drinking much.

What is the most likely cause?

A

Streptococcus

  • This girl has scarlet fever, caused by a streptococcal infection.
  • She has the classic erythematous rash - which is rough and maculopapular.
  • A distinguishing feature is the peri-oral sparing and bright red lips.
115
Q

A 9 month old girl has a widespread rash.

Her mother reports her having 2 days of fevers where she was very unsettled and lethargic.

Then in the last two days a rash has spread from her neck to her torso and upper limbs.

It is erythematous, maculopapular and not pruritic.

A multi-systems examination is normal.

Her mother does not report any fevers today and she is afebrile with normal observations when assessed.

What is the most appropriate course of action?

A

Supportive management

This girl has roseola infantum (6th disease), caused by HHV6 (roseola).

It is generally a benign illness characterised by several days of high temperatures followed by a widespread rash that starts on the torso/neck.

These children tend to be well and do not require intervention.

116
Q

A two year old boy presents to the GP with a swelling in the groin.

His mother has noticed it in the evenings and also when he cries.

On examination there is a small, painless lump on the right just above the scrotum.

The testicles are normal and the spermatic cord is easily palpable.

There is a cough impulse and the contents of the lump are easily reducible.

What is the underlying abnormality?

A

Patent processus vaginalis

This is an inguinal hernia.

They are almost always indirect in children and are caused by failure of the processus vaginalis to obliterate.

They are more common in boys and on the right.

Unreducible hernias are a surgical emergency.

117
Q

An 11-year old Afro-Caribbean boy with a history of sickle cell disease presents to the emergency department with a painful and immobile left leg. There is exquisite tenderness, swelling and erythema over the distal femur. He has a temperature of 39 degrees, is tachycardic and hypotensive. Oxygen saturations are normal, but he is tachypnoeic. Investigations show: haemoglobin 65g/L, WCC 16x109/L, PLT 300x109/L.

What is the most likely causative organism?

A

Salmonella typhi

This patient has sickle cell disease, and is susceptible to Salmonella osteomyelitis.

118
Q

A 7 month old boy has been admitted following a two day history of vomiting.

On further questioning, his parents also report intermittent episodes of inconsolable crying during which he draws his knees up into his chest.

He was born at term and has no known medical history.

Examination is difficult due to his screaming.

However, abdominal distension and blood-stained mucous in his nappy is noted.

What is the most likely diagnosis?

What is the most appropriate investigation to confirm the diagnosis?

A

Abdominal ultrasound

The intermittent abdominal pain (inconsolable crying), vomiting and bloody stool points towards a diagnosis of intussusception.

Intussusception is where one part of the bowel telescopes into the adjacent segment, the trapped portion off bowel can become ischaemic causing abdominal pain.

Abdominal ultrasound will demonstrate the target/doughnut sign - concentric rings of hyper- and hypo- echogenicity.

119
Q

NEONATAL JAUNDICE

Causes <24 hours

Causes 24 hours- 14 days

Causes >14 days (>21 if preterm)

A

Causes <24 hours

  • Haemolytic disorders (Rh incompatibility, ABO incompatibility, G6PD, spherocytosis)
  • Infection (TORCH Screen is indicated)

Causes 24 hours- 14 days

  • Physiological jaundice
  • breast milk jaundice
  • dehydration, infection
  • haemolysis, bruising
  • polycythaemia

Causes >14 days (>21 if preterm)

  • Physiological jaundice
  • breast milk jaundice
  • infection (esp UTI)
  • hypothyroidism
  • G6PD
  • pyloric stenosis
  • bile obstruction (biliary atresia)
  • neonatal hepatitis
120
Q

A 4-year-old girl is brought to the GP by her father as he is concerned that her urine has become dark-red in colour.

On further questioning, he reports that over the last few weeks she has been eating less and less, and has been complaining of pain in her abdomen.

There is no history of recent illness, and she is otherwise fit and well.

Her vital observations are:

  • blood pressure 127/93
  • heart rate 104
  • respiratory rate 20/min
  • temperature 37.9ºC

On examination a mass in the left flank is palpated.

Which of the following is the most likely underlying diagnosis?

A

Nephroblastoma

This is the correct answer. This girl has presented with haematuria, hypertension, fever and an abdominal mass. These signs are consistent with nephroblastoma (Wilms tumour).

121
Q

A 6 week old boy presents to the GP because he has been crying inconsolably all night and this morning his mother noticed he breathed very quickly and rapidly turned blue for a few minutes.

He has otherwise been well, with no fever, vomiting, diarrhoea, cough or coryza. This is the first time this has happened.

He was born at 40 weeks by a spontaneous vaginal delivery.

There were no concerns during the pregnancy and all the antenatal scans were normal.

On general inspection, the baby is now pink with no signs of respiratory distress.

He is alert, irritable, and moving all limbs spontaneously.

The anterior fontanelle is soft.

The lungs are clear to auscultation.

There is a harsh ejection systolic murmur at the left sternal edge.

What is the diagnosis?

What is the treatment?

How does the treatment work?

A

Phenylephrine

Severe Tet spells can be managed with a vaso-constrictive agent (e.g. phenylephrine) as a ‘last line’ medical therapy.

Vasoconstriction will help to increase systemic vascular resistance, reducing the right-to-left shunt and improving cyanosis.